PT76.S2.Q16 - when surveyed about which party they would like to see

lizmiss24lizmiss24 Alum Member
edited January 2016 in Logical Reasoning 50 karma
admin edit: here's the link to the lesson: http://7sage.com/lsat_explanations/lsat-76-section-2-question-16/

Hi everyone,

I understand why (C) is correct for this question, yet I picked (E) during my timed PT and I'm still not 100% sure why it's wrong. From the stim, I understood that a group of respondents was split as follows: 40% wanted conservatives, 40% wanted liberals, and 20% wanted moderates. From that info, we don't know precisely what PERCENTAGE of conservatives/liberals/moderates these respondents each want in the legislature: only that they want them. Maybe one respondent that wanted liberals wanted a legislature with 60% liberals, while another wanted it with 100% liberals. Therefore, this evidence only supports a rough estimate.

The conclusion, however, states that each citizen wants a 40/40/20 split in the legislature, which are EXACT PERCENTAGES. Therefore, I thought that (E) applied (going from a rough estimate to an exact/quantified conclusion.)

Thoughts on this are much appreciated!! :)

Comments

  • lizmiss24lizmiss24 Alum Member
    50 karma
    Anyone? I'd love feedback on this!
  • Q.E.DQ.E.D Alum Member
    edited January 2016 556 karma
    Hi @lizmiss24. The LSAT seems to have tons of questions aimed directly at the fallacy of composition. That's a term for either a) concluding that what's true of a group is true of the members of that group or b) concluding that what's true of the members of a group is true of the group as a whole. This question stim. concludes that since the citizens collectively vote 40% conservative ...etc., then all or most of the citizens individually want those proportions in the legislature. Choice C states the fallacy precisely.

    I'm intrigued by your comments about answer choice E. On the first reading, I saw nothing but the test writers' intentions with this question, but I guess you could reach the conclusion in the stim. arithmetically if you had more data. You apparently noticed that, noted the absence of any such data, and chose E. I commend your thinking there. However, I'd still reject E for these reasons: a) Choice E speaks of differing levels of precision in assumptions v. conclusion, whereas assumptions and conclusion seem to have the same quantitative precision (if not less in the conclusion, which uses a rough notion like 'most'); b) It's not clear the evidence provided supports even rough estimates, as presupposed by choice E; and c) Choice C is true without any qualifications, making it a better fit that E.

    Just my thoughts. Good luck!
  • lizmiss24lizmiss24 Alum Member
    edited January 2016 50 karma
    Hi @Q.E.D. - thanks for your feedback! :) Your reasoning actually helped me a lot.

    I also figure my prior reasoning is wrong because if a respondent wanted the legislature to be 60% Liberal, then they would want the other 40% to be a different party. In that case, you would have to put that respondent in at least two groups, which would result in an overlap and a whole lot of confusion. There's just too many assumptions in there.

    Good luck to you too - we got this!
Sign In or Register to comment.